Here is the problem:
A boy stands at the peak of a hill which slopes downward uniformly at angle $\phi$. At what angle $\theta$ from the horizontal should he throw a rock so that it has the greatest range?
I realize that the same question is posted here: https://physics.stackexchange.com/questions/24235/trajectory-of-projectile-thrown-downhill, but I have some questions that were not answered in that thread:
- Can the problem be solved without a rotation of the coordinate system? If so, how?
- I tried to solve the problem using a rotated coordinate system, but cannot figure out how to finish it (see the work given below).
Here is what I have so far:
- We set up the coordinate system so that the positive $x$ axis coincides with the downward slope of the hill. This simplifies the problem by allowing us to easily relate $\phi$ and $\theta$, through the relation $\alpha = \phi + \theta$.
- $v_{0x} = v_0 \cos\alpha$
- $v_{0y} = v_0 \sin\alpha$
- $a_x=-g \cos(\phi-\frac{\pi}{2})=-g \cos (-(\frac{\pi}{2}-\phi))= g\cos(\frac{\pi}{2}-\phi)=g\sin\phi$
- $a_y=-g \sin(\phi-\frac{\pi}{2})=-g \sin (-(\frac{\pi}{2}-\phi))= -g\sin(\frac{\pi}{2}-\phi)=-g\cos\phi$
- $v_x=v_{0x}+\int_{0}^{t}{a_x(t \prime)dt \prime}=v_0 \cos \alpha+\int_{0}^{t}{(g\sin\phi) dt \prime} =v_0 \cos \alpha + t(g\sin\phi)$
- $x=x_0 + \int_0^t{v_x(t\prime) dt\prime} = \int_0^t{(v_0 \cos \alpha + t\prime(g\sin\phi)) dt\prime}=t(v_0 \cos \alpha) + \frac{1}{2}t^2(g \sin \phi)$
- $v_y=v_{0y}+\int_{0}^{t}{a_y(t \prime)dt \prime}=v_0 \sin \alpha+\int_{0}^{t}{(-g\cos\phi) dt \prime} =v_0 \sin \alpha - t(g\cos\phi)$
- $y=y_0 + \int_0^t{v_y(t\prime) dt\prime} = \int_0^t{(v_0 \sin \alpha - t\prime(g\cos\phi)) dt\prime}=t(v_0 \sin \alpha) - \frac{1}{2}t^2(g \cos \phi)$
- To find the flight time of the projectile, we find the time at which its trajectory intersects the ground (in this case, the $x$ axis), by setting $y=0$ and solving for $t$. $$y=t(v_0 \sin \alpha) - \frac{1}{2}t^2(g \cos \phi)=0$$ $$v_0 \sin \alpha = \frac{1}{2}t(g \cos \phi)$$ $$t=\frac{2v_0 \sin\alpha}{g \cos \phi}$$
- Substituting $t$ into the equation for $x$ gives us the distance traveled by the projectile as a function of the angles $\alpha$ and $\phi$.$$x=t(v_0 \cos \alpha) + \frac{1}{2}t^2(g \sin \phi)$$ $$x=(\frac{2v_0 \sin\alpha}{g \cos \phi})(v_0 \cos \alpha) + \frac{1}{2}(\frac{2v_0 \sin\alpha}{g \cos \phi})^2(g \sin \phi)$$ $$x=\frac{2v_0^2}{g \cos \phi}(\sin \alpha \cos \alpha)+\frac{2v_0^2}{g \cos \phi}(\sin^2\alpha \frac{\sin\phi}{\cos\phi})$$ $$x=\frac{2v_0^2}{g \cos \phi}(\sin\alpha\cos\alpha+\sin^2\alpha\tan\phi)$$
- I noticed that the solution in the other thread proceeds from here by differentiating $x$ with respect to $\alpha$, holding $\phi$ constant, which gives $$\frac{dx}{d\alpha}=\frac{2v_0^2}{g \cos \phi}(\frac{d}{d\alpha}(\frac{1}{2}(\sin(2\alpha)+\sin^2\alpha\tan\phi))$$ $$\frac{dx}{d\alpha}=\frac{2v_0^2}{g \cos \phi}(\cos(2\alpha)+2\sin\alpha\cos\alpha\tan\phi)$$ $$\frac{dx}{d\alpha}=\frac{2v_0^2}{g \cos \phi}(\cos(2\alpha)+\sin(2\alpha)\tan\phi)$$ This equation allows us to examine how $x$ changes with respect to $\alpha$. We see that $x$ increases as $\alpha$ increases, up to a certain point, and then decreases as $\alpha$ increases past this value. This means that the graph of $x$ has a relative maximum at the value of $\alpha$ which produces the maximum range.
- We want to find the value of $\alpha$ that results in the maximum range of the projectile. In other words, we must determine the value of $\alpha$ for which the graph of $x$ has a relative maximum. We achieve this by setting $$\frac{dx}{d\alpha}=0=\frac{2v_0^2}{g \cos \phi}(\cos(2\alpha)+\sin(2\alpha)\tan\phi)$$ Dividing each side by $\frac{2v_0^2}{g \cos \phi}$ produces $$\cos(2\alpha)+\sin(2\alpha)\tan\phi=0$$ Here is where I get lost. It seems like this should be the easy part, because the only thing left to do is solve the above equation for $\alpha$, but I don't know how to do it. Could someone please explain this part to me?
Additionally, I would like to know whether the problem can be solved without rotating the coordinate system. I originally set out to solve it using the standard rectangular coordinate system, but got bogged down in some equations that seemed to lead nowhere. Thanks for your help.
No comments:
Post a Comment